Integument/Dermatology Flashcards

1
Q

A 61-year-old male farmer presents to your clinic because he is concerned about a skin lesion on his face. The lesion is shown in the accompanying photograph. Which one of the following is the most likely diagnosis?

A. Keratoacanthoma.
B. Basal cell carcinoma.
C. Implantation dermoid cyst.
D. Amelanotic malignant melanoma.
E. Squamous cell carcinoma.

A

B. Basal cell carcinoma.

The appearance of the lesion is consistent with nodular basal cell carcinoma (BCC). BCC is the most common skin cancer. Unlike squamous cell carcinoma (SCC), BCC can be seen in non-photoexposed areas as well. Nodular BCC is the most common subtype. The pearly appearance, shiny surface and telangiectasis are characteristic features. Additionally, the lesion in the photograph has undergone ulceration at parts.

Option A: Keratoacanthoma has the central crater filled with necrotic tissue, which is not seen here.

Option C: Implantation dermoid cysts are firm subdermal cystic nodules that often occur following a penetrating trauma. The fingertips of mechanics are the most common site for such lesions.

Option D: Amelanotic melanoma can resemble a nodular BCC; although it will lack the pearly appearance of BCC, which can be noted in this photograph.

Option E: Squamous cell carcinoma does not fit the clinical picture. It usually presents with an ulcerating plaque.

How well did you know this?
1
Not at all
2
3
4
5
Perfectly
2
Q

A 43-year-old farmer from Queensland presents with a pigmented lesion on his upper chest, shown in the accompanying photograph. The lesion appeared 4 months ago, but recently has enlarged. The lesion has been itchy and oozing and painful for the past 3 weeks. Which one of the following is the most likely diagnosis?

A. Malignant melanoma.
B. Keratoacanthoma.
C. Basal cell carcinoma.
D. Blue nevus.
E. Benign pigmentation.

A

A. Malignant Melanoma.

Although the lesion shown in the photograph may be mistakenly considered as a pigmented nodular basal cell carcinoma, a more careful inspection of the lesion indicating pigmentation of different colors as well as the history of the rapid progression over 4 months suggests malignant melanoma (nodular type) as the most likely diagnosis.

Option B: Keratoacanthoma has the typical crater which is absent here. The crater often has keratin or dead skin cells inside.

Option C: Pigmentation and color variegation of this lesion favors melanoma rather than basal cell carcinoma. Moreover, the rapid progression and evolution of symptoms such as itching and tenderness are more commonly seen in melanoma.

Option D and E: Clinical presentation and the history (rapid course of progression and evolving of size and symptoms) definitely exclude blue nevus and benign pigmentation as the likely diagnoses.

Kerathocanthoma
How well did you know this?
1
Not at all
2
3
4
5
Perfectly
3
Q

On a routine health examination of a 72-year-old man, you notice lesions depicted in the following photograph on his back. Which one of the following could be the most likely diagnosis?

A. Superficial spreading melanoma.
B. Mycosis fungoides.
C. Hutchinson melanotic freckles.
D. Seborrheic keratosis.
E. Solar keratosis.

A

D. Seborrheic keratosis.

The appearance of the lesions are suggestive of seborrheic keratosis. Characteristic features include variegated pigmentation from black to tan, the verrucous greasy surface and occurrence in crops.

Seborrheic keratoses are well-defined benign lesions which are often pigmented and can occur on the head, neck, trunk and limbs and areas not always significantly exposed to sunlight. Although the etiology is unknown, sun exposure may play a role.

They usually start to appear in the third decade of life and increase in number as the age advances. Seborrheic keratoses are the most common benign pigmented tumors in those aged 50 years and over.

Seborrheic keratoses are typically round plaques with adherent greasy scale. They seem to be stuck on the skin. Their color may vary from black to tan. The surface appears velvety to verrucous. Sometimes they are so verrucous that may be mistaken with warts, particularly if not heavily pigmented. Sometimes there are surface cracks and horn cysts.

Some seborrheic keratoses are flat. Flat lesions, particularly those of the face and the limbs, may be difficult to be distinguished from melanoma or lentigo maligna. This is even more true if the lesions are inflamed and consequently, enlarged and itchy with tendency to bleed.

Option A: Superficial spreading melanoma is another likely diagnosis, especially if the lesions were more superficial. The large number of the lesions, smooth margins and the greasy warty look of the lesion in the center make seborrheic keratosis a more likely diagnosis. Biopsy and histological studies are required if the diagnosis is in doubt.

Option B: Mycosis fungoides is a misnomer describing a rare involvement of the skin with lymphoma. The appearance would be irregular eczematous dermatitis progressing to form plaques.

Option C: Hutchinson melanotic freckles have more irregular borders and are more flat than the lesions depicted. Furthermore, Hutchinson melanotic freckle more commonly tends to be solitary and on sun-exposed areas.

Option E: Solar keratoses are firm, hyperkeratinous plaques of 2 to 10mm in size seen in sun-exposed areas e.g. dorsum of hands.

How well did you know this?
1
Not at all
2
3
4
5
Perfectly
4
Q

A 41-year-old farmer has presented with a lesion on his right sole. The lesion appeared 4 months ago and has progressively enlarged since then. The lesion is shown in the accompanying photograph. Which one of the following could be the most likely diagnosis?

A. Neuropathic ulcer.
B. Malignant melanoma.
C. Simple nevus.
D. Pigmentation from heat.
E. Squamous cell carcinoma.

A

B. Malignant Melanoma.

The size and color of the lesion, the irregularity, and the rapid progression are highly suggestive of malignant melanoma.

Option A: Neuropathic ulcers occur over pressure points of areas with impaired sensation due to conditions such as diabetes mellitus (the most common cause), syphilis, leprosy, and other neuropathies. They present a completely different picture.

Option C: The rapid progression, size, and irregularity are consistent with melanoma rather than simple nevus as a diagnosis. A simple nevus usually has regular borders, is often unicolor, and does not spread so fast.

Option D: Pigmentation from heat, also called ‘erythema ab igne’ is tanning of the skin, where it is chronically exposed to heat.

Option E: Cutaneous squamous cell carcinoma (SCC) presents with a usually amelanotic skin lesion in sun-exposed areas.

How well did you know this?
1
Not at all
2
3
4
5
Perfectly
5
Q

Which one of the following is not correct regarding contact dermatitis?

A. Plants are the most common cause of contact dermatitis.
B. It can be treated with topical steroids.
C. It can be treated with oral steroids.
D. The application of skin moisturizers can assist the healing process.
E. Avoidance of the allergen is essential to the prevention of further flare-ups.

A

A. Plants are the most common cause of contact dermatitis.

Allergic contact dermatitis is a type 4 or delayed hypersensitivity reaction and occurs 48–72 hours after exposure to the allergen. The mechanism involves CD4+ T-lymphocytes and the release of cytokines that activate the immune system and cause dermatitis.

Contact allergy occurs predominantly from an allergen on the skin rather than from internal sources or food. Of note, a patient might have been in contact with the allergen for years without it causing dermatitis. Even small quantities of an allergen can induce dermatitis.

Allergic contact dermatitis is especially common in metal workers, hairdressers, beauticians, healthcare workers, cleaners, painters, and florists. Patients with impaired barrier function are more prone to contact dermatitis.

Allergic contact dermatitis develops hours after contact with the responsible material and settles down over some days provided the skin is no longer in contact with the allergen. The condition is often limited to the site of contact with the allergen, but it may extend outside the contact area or become generalized. Transmission from the fingers can lead to dermatitis on the eyelids and genitals. Dermatitis is unlikely to be due to a specific allergen if the area of skin most in.

Some typical examples of allergic contact dermatitis include:
* Eczema in the skin in contact with jewelry items, due to contact allergy to nickel
* Reactions to fragrances in perfumes and household items
* Eczema under adhesive plaster, due to contact allergy to rosin
* Swelling and blistering of face and neck in reaction to permanent hair dye, due to allergy to paraphenylenediamine
* Hand dermatitis caused by rubber gloves
* Itchy red face due to contact with methylisothiazolinone, a preservative in wash-off hair products and baby wipes
* Fingertip dermatitis due to acrylates used in hair extensions and nail cosmetics.
* Reactions after dental implants containing acrylates
* Localized blistering at the site of topical medications
* Swelling and blistering on exposed sites (e.g., face and hands) due to contact with plants such as poison ivy

Therefore, prevention of further contact with the allergen is the mainstay of the treatment. A skin patch test may be required to find the culprit. Active lesions are often treated with topical steroids (option B). More severe cases may necessitate a short course of oral steroids (option C).

Depending on the severity and the clinical course, other treatment options can include phototherapy/photochemotherapy, immune suppressants such as azathioprine or ciclosporin, or immune system modulators tacrolimus ointment.

The application of skin moisturizers (option D) can promote general skin health conditions, alleviate the symptoms, and accelerate the healing process.

Although plants could result in contact dermatitis, they are not the most common cause of contact dermatitis. Nickel and other metals/ chemicals are the most common cause of contact dermatitis.

How well did you know this?
1
Not at all
2
3
4
5
Perfectly
6
Q

A 32-year-old woman presents to your clinic for a skin checkup. On examination, she is found to have a skin lesion on her upper back. The lesion is highly suspected of being melanoma. An excisional biopsy is performed with 2 mm margins and the specimen is sent to a laboratory for histopathological studies. The result is a melanoma of 2.5 mm in depth. The margins, however, are clear. Which one of the following would be the next best step in management?

A. A wider excision.
B. Radiation therapy.
C. Sentinel node biopsy.
D. CT scan of the head.
E. Chemotherapy.

A

A. A wider excision.

Every lesion suspected of being melanoma should be surgically excised with 2 mm margins both as the initial management and the most appropriate step in diagnosis. Further management is then guided by the results of the biopsy. If the lesion is proved to be melanoma, a wider excision is needed. The margins of the second excision depend on the reported depth of melanoma, and is according to the following table.

For melanomas greater than 1mm in depth, a sentinel node biopsy is recommended during the second excision to assess the potential metastasis to the subcutaneous tissue and lymph nodes.

How well did you know this?
1
Not at all
2
3
4
5
Perfectly
7
Q

Which one of the following is the most important prognostic factor in basal cell carcinoma?

A. Depth of the lesion.
B. The colour of the lesion.
C. Residual cancer cells in the margins of the excised lesion.
D. The width of the lesion.
E. The site of the lesion.

A

C. Residual cancer cells in the margins of the excised lesion.

Generally, the prognosis for patients with BCC is excellent, with a 100% survival rate for cases that have not spread to other sites. Typically, basal cell tumors enlarge slowly and relentlessly and tend to be locally destructive. Periorbital tumors can invade the orbit, leading to blindness, if diagnosis and treatment are delayed. BCC arising in the medial canthus tends to be deep and invasive and more difficult to manage; this type of BCC can result in perineural extension and loss of nerve function.

Although BCC is a malignant tumor, it rarely metastasizes. The incidence of metastatic BCC is estimated to be less than 0.1%. The most common sites of metastasis are the lymph nodes, lungs, and bones.

Treatment of BCC is curative in more than 95% of cases; however, BCC may recur, especially in the first year, or develop in new sites. The prognosis of BCC is, therefore, mostly based on the likelihood of recurrence.

There are several prognostic factors affecting the chance of recurrence of a BCC. Of which, the clinical location, the architectural pattern and excision margins are the most important factors. Of these three, most reports consider the presence or absence of tumor cells in the excision margins as the most significant prognostic factor regarding recurrence.

Option A and D: Although the width and depth of tumors are important, as long as they can be excised with tumor-free margins the prognosis is good with recurrence being less likely.

Option C: Although different BCCs can vary in color and some with specific morphological or histological characteristics more likely to recur, color alone is not of great significance in determining the prognosis.

Option E: The site of the lesion is important due to the fact that lesions in specific areas are less likely to be excised with tumor-free margins. Some of these areas are nose, eyelids, temples, pre- and post-auricular regions and lower legs.

TOPIC REVIEW
The following parameters affect, to different extends, the outcome and prognosis of BCC:
* Recurrent tumors (poorer prognosis)
* Multiple tumors (poorer prognosis)
* Size and depth of invasion (stage)
* Morphemic, infiltrating and micronodular (poorer prognosis) Morphological and histological subtype
* Treatment modality (Mohs surgery has been associated with best prognosis)
* Incomplete excision (probably the poorest prognosis)
* Perineural spread
* Nevoid basal cell carcinoma syndrome (poorer prognosis)
* Special sites (poorer prognosis):
Nose
Eyelids
Temple
Pre- and post-auricular
Lower legs

How well did you know this?
1
Not at all
2
3
4
5
Perfectly
8
Q

A 59-year-old farmer presents to your clinic with a lesion on his upper chest. The lesion has appeared and progressively enlarged in the past 2 months. Examination establishes a diagnosis keratoacanthoma with high certainty. Keratoacanthoma is more likely to be confused with which one of the following?

A. Basal cell carcinoma.
B. Squamous cell carcinoma.
C. Pyogenic granuloma.
D. Seborrheic keratosis.
E. Granuloma fissuratum.

A

B. Squamous cell carcinoma.

Keratoacanthoma (KA) is a rapidly growing tumor of keratinocytes. They are almost exclusively seen in sun-exposed areas. The characteristic feature is the crater; the central part of the lesion is necrotic, giving the lesion the appearance of a volcano.

KAs are now considered a low-risk variant of squamous cell carcinoma (SCC). The major diagnostic problem is confusion with SCC, especailly for KAs of the nose and the lips. Interestingly, sometimes KAs cannot be told apart from SCCs based on cytological studies and the whole specimen is needed for differentiation.

Option A: An ulcerated nodular BCC may resemble KA, but SCC remains the most confusing differential diagnosis.

Option C and D: Pyogenic granuloma and seborrheic keratosis are very unlikely to be confused with KA.

Option E: Granuloma fissuratum is a firm red fissured fibrotic granuloma found in the gums and buccal mucosa. It is usually caused by ill-fitting dentures.

How well did you know this?
1
Not at all
2
3
4
5
Perfectly
9
Q

A 32-year-old female patient presents to your practice complaining of a skin lesion on the ventral aspect of her right forearm, which has developed during the past 6 weeks. The lesion is shown in the accompanying photograph. She had a successful renal transplant last year. Which one of the following would be the most appropriate treatment?

A. No active treatment is needed,as it resolves spontaneously.
B. Surgical removal of the lesion.
C. Radiotherapy.
D. Topical podophyllin.
E. Removal with liquid nitrogen.

A

B. Surgical removal of the lesion.

The lesion in the photograph is a domed nodule with a necrotic plug in the center. The appearance is characteristic of Keratoacanthoma (KA). KAs are keratinizing skin tumors which grow more rapidly (6-8 weeks) compared with basal cell carcinoma, squamous cell carcinoma and melanoma. They are usually seen as a solitary nodule in sun-exposed areas. It often develops later in life with a predilection for women.

If the lesion is left untreated, spontaneous healing and resolution may occur within 3 to 6 months; however, at instances it may continue to grow or even metastasize.

Since KA is clinically indistinguishable from malignant lesions, especially squamous cell carcinoma, the preferred management would be the same as for squamous cell carcinoma which is elliptical surgical excision with margins of 3-5mm (the same that would be done for squamous cell carcinoma of the skin).

KAs share features such as infiltration and cytological atypia with SCCs; hence they are considered to be a variant of SCC called SCC-KA type.

Although a shave biopsy may be used for diagnosis, it is not an adequate final treatment and complete excision should eventually follow.

The need for complete surgical removal is even more in patients who have undergone organ transplantation, because these patients are on immunosuppressive medications drugs; therefore, the lesion is more likely to be malignant. Even if the case is KA, spontaneous resolution is far less likely in the presence of immunosuppression.

How well did you know this?
1
Not at all
2
3
4
5
Perfectly
10
Q

A 36-year-old woman presents to your clinic concerned about a lesion on her right shin. The lesion appeared several days after the site was stung by a bee. On examination, there is a 0.6mm nodule on the lateral aspect of the right shin as illustrated in the photograph. It is not tender to touch. Which one of the following could be the most likely diagnosis?

A. Basal cell carcinoma.
B. Squamous cell carcinoma.
C. Dermatofibroma.
D. Molluscum contagiosum.
E. Pyogenic granuloma.

A

C. Dermatofibroma.

The appearance, history, and exam findings are suggestive of dermatofibroma as the most consistent diagnosis.

Dermatofibroma, also called sclerosing hemangioma or histiocytoma, is a common pigmented nodule in the dermis due to the proliferation of fibroblasts, usually following minor trauma.

Dermatofibroma is more commonly seen in women on the lower leg. The lesion is a button-like nodule that is firm and well-circumscribed. The size may vary from 0.5 to 1cm. It can be pink, brown, tan, gray, or violaceous. The nodule is freely mobile over the deeper structures. The characteristic feature on examination is a dimpling of the nodule when it is pinched laterally (dimple sign). The lesion is often asymptomatic but may be itchy or tender.

Other options have different characteristic features.

How well did you know this?
1
Not at all
2
3
4
5
Perfectly
11
Q

A 47-year-old man presents because of a lesion on the back of his right shoulder. The lesion is illustrated in the following photograph. Which one of the following is the most appropriate advice?

A. The lesion should be excised, as it is malignant and can extend locally, but not through lymphnodes.
B. The lesion is benign and does not need to be excised.
C. The lesion is benign, but should be excised because it can become malignant.
D. The lesion is benign, but should be excised because it can become infected.
E. The lesion is malignant and should be excised because it can metastasize through adjacent lymph nodes.

A

D. The lesion is benign, but should be excised because it can become infected.

The photograph shows a well-circumscribed lesion, which appears to arise from within the epidermis. It also has a punctum in the center. These features are characteristic of an epidermoid (sebaceous) cyst.

Epidermoid cysts originate from the dermis and are attached to the skin. Since they are related to the pilosebaceous follicle, they can occur in any hair-bearing region such as the scalp or scrotum.

Of note, the yellow cheesy material within the cyst is desquamated keratin, not sebum.

Epidermoid cysts are benign and do not progress to malignancy; however, it is recommended that they be removed, as superinfection may lead to suppuration and abscess formation.

How well did you know this?
1
Not at all
2
3
4
5
Perfectly
12
Q

A 6-year-old boy is brought to you by his mother because she is concerned about skin lesions on the boy’s back and trunk. The lesions have started to appear and increase in number for the past week. The rash over his back is shown in the accompanying photograph. On examination, the child is well and healthy with no other abnormal findings. Which one of the following is the most likely diagnosis?

A. Varicella zoster.
B. Impetigo.
C. Molluscum contagiosum.
D. Herpes simplex.
E. Papilloma virus (warts).

A

C. Molluscum contagiosum.

The lesions in the photograph are pearly dome-shaped papules with central umbilication characteristic of molluscum contagiosum as the most likely diagnosis. Molluscum contagiosum is a common viral infection of childhood caused by molluscipoxvirus a member of the poxvirus family.

The infection presents with firm, smooth, spherical papules that are pearly white and have a central dimple (umbilicus). Most papules range from 1 to 3 mm; however, lesions of up to 1-2 cm have been reported mostly due to coalescing smaller lesions.

Lesions can develop anywhere, but flexures and areas of friction are more frequently involved. Lesions may also occur in the anogenital area and are not usually associated with sexual abuse of the child. Involvement of the eyelid margins may lead to chronic conjunctivitis.

Molluscum contagiosum has a benign course and nature. Most patients experience the spontaneous resolution of the lesions within 3 to 6 months, but on occasion, it may take up to 3 years.

Option A: Varicella zoster infection in children (chickenpox) presents with blisters at different stages (intact, ruptured, crusted, or sometimes infected) which are often itchy. The eruption usually follows prodromal symptoms such as malaise and fever. None of the history and physical examination features is consistent with varicella zoster infection.

Option B: Impetigo is a superficial skin infection presenting with lesions often with honey-colored crusting.

Option D: Herpes simplex presents with painful vesicles that may become unroofed to produce a raw appearance.

Option E: Papillomavirus infection (wart) presents a completely different picture. Often it is cauliflower-like with a rough surface.

How well did you know this?
1
Not at all
2
3
4
5
Perfectly
13
Q

There are a number of skin lesions which are related to cumulative sun-exposure. Which one of the following is most likely to be caused by chronic sun exposure?

A. Actinic (solar) keratosis.
B. Junctional nevus.
C. Seborrheic keratosis.
D. Tinea versicolor.
E. Keratoacanthoma.

A

A. Actinic (solar) keratosis.

Actinic keratosis and Bowen’s disease are seen frequently in light-skinned individuals, who have had significant sun-exposure. They are precancerous lesions for squamous cell carcinoma.

Option B: Junctional nevus is not associated with sun-exposure.

Option C: Seborrheic keratosis does not seem to have strong association with sun-exposure because it frequently appears in areas not exposed to and affected by the sunlight.

Option D: Tinea versicolor is a skin infection caused by Malassezia furfur and is aggravated by heat and damp, but not related to sun exposure.

Option E: Keratoacanthoma is a rather benign tumor arising from pilocebaceous glands, mostly in sun-exposed areas such as face. The association with sun-exposure is not as significant compared with actinic keratosis or Bowen’s disease.

Actinic Keratosis
How well did you know this?
1
Not at all
2
3
4
5
Perfectly
14
Q

A 50-year old farmer presents to your practice with a dark mole on his left cheek. The mole has been there for 20 years, but has enlarged and become slightly lumpy and itchy over the past 4 months. Which one of the following would be the most appropriate management option for this patient?

A. Treat the lesion using liquid nitrogen.
B. Ask the patient to return for review in 3 months.
C. Remove the lesion using the laser.
D. Use topical imiquimod for 6 weeks.
E. Excisional biopsy of the lesion for histopathology.

A

E. Excisional biopsy of the lesion for histopathology.

A changing mole, a mole with an irregular border, bleeding, itching, or color variegation is melanoma until proven otherwise. When melanoma is suspected, the next best step is an excisional biopsy of the lesion with 2 mm margins. Elliptical incisions are made so that reconstruction of the wound is more straightforward. If histopathological results confirm the diagnosis, referral to a plastic surgeon is necessary for a wider excision with margins of 5 mm to 3 cm depending on the depth of the lesion.

NOTE - in GP settings, patients with suspected lesions on sensitive areas such as the head and neck should be referred to a plastic surgeon even for the initial excision. If the referral was an option, it would be the correct answer.

How well did you know this?
1
Not at all
2
3
4
5
Perfectly
15
Q

Which one of the following is the most important risk factor for melanoma?

A. Family history of basal cell carcinoma or squamous cell carcinoma of the skin.
B. A cousin with melanoma in family history.
C. Working outdoors since the age of 18.
D. Multiple sunburn since childhood.
E. Presence of solar keratosis.

A

D. Multiple sunburns in childhood.

The following table classifies risk factors for developing melanoma in a descending order are as follows.

Among the given options multiple sunburns in childhood predispose to the most significant risk (it is associated with only slightly elevated risk factor, yet the greatest among other options).

Option A: A family history of non-melanoma skin cancers (NMSC) is an important risk factor for NMSC but not for melanoma; however, personal history of NMSC is a significant risk factor for melanoma.

Option B: Family history of melanoma is not that significant if not in the first-degree relatives (parents, siblings).

Option C: Working outdoors since the age of 18 years is another risk factor, but not as important as multiple sunburns.

Option E: Solar keratosis is a significant risk factor for squamous cell carcinoma, not melanoma.

How well did you know this?
1
Not at all
2
3
4
5
Perfectly
16
Q

The mother of a 5-year-old boy, who has recently undergone removal of a melanoma on her back asks you what can play a major role in developing melanoma in her child in the future. Which one of the following options would be your answer to her question?

A. Family history of melanoma.
B. Sunburn.
C. Fair skin.
D. UV exposure.
E. The presence of multiple dysplastic moles.

A

E. The presence of multiple dysplastic moles.

The following table categorizes the risk factors for developing melanoma in a descending order of significance.

All the given options are potential risk factors for developing melanoma, but the presence of multiple moles (>100 nevi or more than 5 dysplastic nevi) is the most significant risk factor.

Red hair and blue eyes are associated with a highly- and moderately-increased risk of melanoma respectively. Fair complexion is another risk factor, but the degree of association with melanoma depends on the age of the patient. In patients over the age of 45 years, fair skin can be as significant a risk factor as multiple nevi.

UV exposure and sunburns are associated with a slightly increased risk of melanoma. In sun-related melanomas, acute intense and intermittent blistering sunburns, especially on areas of the body that only occasionally receive sun exposure, are the most significant risk factors for the development of sun exposure-induced melanoma. This sun-related risk factor for malignant melanoma differs from squamous or basal cell carcinomas.

NOTE - lentigo malignant melanoma (LMM) is an exception to this rule because it frequently appears on the head and neck of older individuals who have a history of long-term sun exposure; therefore, prolonged sun (UV) exposure is the greatest risk factor for LMM.

Exposure to ultraviolet radiation (UVR) is a critical factor in the development of sun-related melanomas. Ultraviolet A (UVA), wavelength 320-400 nm, and ultraviolet B (UVB), 290-320 nm, potentially are carcinogenic and can attribute to melanoma induction. The suggested mechanisms through which UV exposure can play its role are:
* Suppression of the immune system of the skin
* Induction of melanocyte cell division
* Free radical production
* Damage to melanocyte DNAc

Interestingly, melanoma does not have a direct relationship with the amount of sun exposure because it is more common in white-collar workers than in those who work outdoors. Also, it is more common on the back of men’s and women’s lower legs when there is no significant sun exposure.

How well did you know this?
1
Not at all
2
3
4
5
Perfectly
17
Q

On a routine health examination, the lesion shown in the following photograph is found on the inner side of the lower lip of a 37-year-old man. The lesion is painless. Which one of the following is the most likely diagnosis?

A. Sebaceous cyst.
B. Peutz-Jegher’s syndrome.
C. Squamous cell carcinoma of the lip.
D. Mucous cyst.
E. Basal cell carcinoma.

A

D. Mucous cyst.

The lesion shown has a bluish-glistening color and is dome-shaped. These are characteristics of a benign mucoid cyst. A mucous cyst, also known as a mucocele forms when mucus or saliva escapes into surrounding tissues. A lining of granulation or connective tissue is formed to create a smooth, soft round fluid-filled lump. They most commonly occur on the inner surface of the lower lip (75% of cases) but may also appear on the floor of the mouth or on the gums, buccal mucosa, and tongue. If persistent or bothersome, an incision and evacuation of the cyst are performed.

Option A: Sebaceous cysts are caused by obstruction of a sebaceous gland in hair-bearing skin. It does not occur in the lip.

Option B: Peutz - Jegher’s syndrome is associated with melanocytic spots on the buccal mucosa and in the gastrointestinal tract.

Option C and E: Both squamous cell carcinoma (SCC) and basal cell carcinoma (BCC) of the lip occur at the mucocutaneous junction of the lip. The mucus membrane of the lip is unlikely to be the site an SCC or BCC arises. Moreover, patients with SCC or BCC typically are older.

Peutz - Jegher's syndrome
How well did you know this?
1
Not at all
2
3
4
5
Perfectly
18
Q

Which one of the following lesions of squamous epithelium is not premalignant?

A. Solar (actinic) keratosis.
B. Leukoplakia
C. Intradermal nevus.
D. Bowen’s disease.
E. Chronic radiation dermatitis.

A

C. Intradermal nevus.

Of the given options, the only one not associated with skin cancers is intradermal nevus. Benign melanocytic nevi have three major types, classified according to the position of the melanocytes in relation to epidermal:
* Intradermal nevus: all of the nevus cells are within dermis. This type accounts for a majority of benign congenital nevi.
* Junctional nevus: the nevus cells are located at the junction of the basal epidermal layers and dermis.
* Combined nevus: nevus cells are both intradermal and junctional.

Option A: Solar (actinic) keratosis is usually a raised plaque usually on sun-exposed area of the skin. It is a precursor of SCC.

Option B: Leukoplakia is whitish lesion in the oral cavity and associated with SCC.

Option D: Bowen disease is considered SCC in situ, and is malignant.

Option E: Chronic radiation dermatitis may result in SCC.

How well did you know this?
1
Not at all
2
3
4
5
Perfectly
19
Q

Which one of the following skin malignancies is most likely to arise from a burn scar?

A. Basal cell carcinoma.
B. Malignant melanoma.
C. Squamous cell carcinoma.
D. Fibrosarcoma.
E. Sweat gland adenocarcinoma.

A

C. Squamous cell carcinoma.

The most common skin malignancy arising from a burn scar is squamous cell carcinoma (SCC). Marjolin ulcer is a less common type of SCC of the extremities found on chronic ulcers or burn scars. Marjolin ulcers occur on average approximately 30 years after an injury to the skin that results in a scar or an ulcer (range 10–75 years). Rarely, an acute Marjolin ulcer may develop between 6 weeks and 1 year of injury. It is estimated that around 2% of thermal burns scars turn into Marjolin ulcers.

Marjolin ulcer can affect people of all ages, most commonly between 40 and 60 years of age. Men are 2–3 times more likely be diagnosed with Marjolin ulcer than women. All races and skin types can develop Marjolin ulcers. The most common sites for Marjolin ulcers are the legs and feet. The ulcers can also form on the head and neck.

Other options are malignancies not seen or very rarely seen in areas with a burn scar.

Marjolin Ulcer
How well did you know this?
1
Not at all
2
3
4
5
Perfectly
20
Q

Which one of the following is the most likely diagnosis of the lesion shown in the accompanying photograph?

A. Keratoacanthoma.
B. Basal cell carcinoma.
C. Seborrheic dermatitis.
D. Malignant melanoma.
E. Simple nevus.

A

D. Malignant Melanoma.

The photograph shows a pigmented lesion on the left side of the nose. With pigmentation, the two most likely diagnoses are simple nevi or malignant melanoma. Early melanomas may be differentiated from benign nevi by the ABCD:
* A - Asymmetry
* B - Border irregularity
* C - Color that tends to be very dark black or blue and variable
* D - Diameter ≥ 6 mm

The lesion is asymmetrical (A) and has irregular borders (B). It shows variegation (variation in colors) (C) and seems to be larger than 6mm; therefore, melanoma seems to be the most likely diagnosis.

Keratoacanthoma, basal cell carcinoma, and seborrhoeic dermatitis present quite differently.

How well did you know this?
1
Not at all
2
3
4
5
Perfectly
21
Q

A 52-year-old golfer man presents to your GP clinic with an ulcer on his right pinna. He says the lesion is itchy and easily bleeds on scratching. On examination, deeply sunburned areas around the lesion are noted. The lesion is shown in the accompanying photograph. Which one of the following can be the most likely diagnosis?

A. Basal cell carcinoma.
B. Squamous cell carcinoma.
C. Malignant melanoma.
D. Bowen’s disease.
E. Keratoacanthoma.

A

B. Squamous cell carcinoma.

The photograph shows a flat amelanotic lesion on the left pinna that has undergone ulceration and is slightly crusted. The appearance of the lesion is consistent with cutaneous squamous cell carcinoma (SCC) as the most likely diagnosis.

The classic presentation of a cutaneous SCC includes a shallow ulcer with heaped-up edges, often covered by a plaque usually in a sun-exposed area. Typical surface changes may include the following:
* Scaling
* Ulceration
* Crusting
* A cutaneous horn

Less commonly, cutaneous SCC presents as a pink cutaneous nodule without overlying surface changes. Regional spread of head and neck cutaneous SCCs, may result in enlarged preauricular, submandibular, or cervical lymph nodes.

NOTE - Although, the appearance of the lesion resembles SCC, it should be noted that at times SCC, keratoacanthoma, BCC, or even amelanotic forms of melanoma may look similar and the definite diagnosis cannot be made unless biopsy and histologic studies are performed. However, because of the classic features of the lesion and also the fact that squamous SCC is the most common skin cancer, SCC would be the most likely diagnosis in this case.

Option A: Although basal cell carcinoma (BCC) of this area is more common than SCC, the characteristic features of BCC, especially in its most common form – nodular, is different.

Option C: Malignant melanoma is often pigmented. This amelanotic lesion is less likely to be melanoma.

Option D: Bowen’s disease (SCC in situ) presents with an asymptomatic well-demarcated erythematous patch or plaque. The presence of symptoms makes Bowen’s disease a less likely yet possible diagnosis. Moreover, ill-defined borders of this lesion is not in favor of Bowen’s disease.

Option E: Keratoacanthoma often has a nodular structure with central crater which is absent here.

How well did you know this?
1
Not at all
2
3
4
5
Perfectly
22
Q

Which one of the following statements is correct regarding nail apparatus melanoma?

A. It is a common form of melanoma.
B. It has a good prognosis.
C. The 5-year survival depends on the depth (thickness) of the lesion.
D. Nail removal may be curative.
E. There is a strong association to sun exposure.

A

C. The 5-year survival depends on the depth (thickness) of the lesion.

Nail apparatus melanomas (subungual melanoma) are rare but frequently fatal neoplasms. They account for between 1.5-3% of all melanomas. All age groups can be affected; however, it is most common in the 7th decade of life.

Melanoma of the nail initially presents as a longitudinal pigmented streak. Later, the proximal nail fold may become pigmented (Hutchinson’s sign). As the disease advances, the nail plate may be destroyed. By this stage, the disease is often advanced, with lymph node metastasis frequently present.

As with other melanomas, increased tumor depth and the presence of ulceration are associated with a worse prognosis and decreased five-year survival.

70% of subungual melanomas occur on the thumb or great toe. Subungual melanomas involving the toes can be managed easily with digital amputation at the metatarsal-phalangeal joint. If the first toe (big toe) is involved, complete amputation should be avoided when the lesion is amenable to less invasive oncological treatments because of the importance of the toe in balance.

Whenever possible, subungual melanomas of the fingers should be resected at the distal interphalangeal joint to preserve function. Melanomas located more proximally on the fingers can often be managed with wide local excision of soft tissue, skin grafts, or local flaps for soft tissue coverage.

NOTE -Longitudinal melanonychia occurs in other conditions, including benign nevi of the nail matrix and the multiple linear streaks that are common in people with dark skin. However, patients with a solitary, widening, irregularly shaped, or irregularly pigmented streak in the nail should undergo a biopsy of the nail matrix to evaluate for melanoma.

Option A: Nail apparatus melanoma accounts for 1.5-3% of all melanomas. It is not the most common form of melanoma.

Option B: Nail apparatus melanoma has a poor prognosis.

Option D: Nail removal has no role in the management of melanoma, as the lesion is in the subungual matrix.

Option E: Nail apparatus melanomas occur with almost equal incidence in people with black skin, Asians, and Caucasians, and in people in tropical versus nontropical climates, sun exposure does not seem to be an etiology.

https://www.racgp.org.au/afp/2016/november/pigmented-lesions-of-the-nail

Hutchinson's Sign
23
Q

A 42-year-old male, accountant by profession, comes for screening and risk assessment for melanoma. Which one of the following, if present in the history, would be of highest risk for developing melanoma?

A. Sunburns in childhood.
B. Irish background.
C. Family history of basal cell carcinoma.
D. A cousin who has recently been diagnosed with melanoma.
E. Working outdoors in early adulthood.

A

A. Sunburns in childhood.

The following table categorizes risk factors for developing melanoma in a descending order of significance.

Of the given options, multiple sunburns in childhood are the most significant risk factor for melanoma. Of UV-related risk factors, sunburns in the first and second decades of life, especially if acute and blistering, carries the most significant risk factor compared with other options, but not in general because UV exposure is associated with only a slightly increased risk of developing melanoma (relative risk ~1.5).

Option B and E: Irish background (Caucasian) and working outdoors since early adulthood may be associated with slightly increased risk, but not as significant as sunburns in childhood.

Option C: A family history of non-melanoma skin cancers (NMSC) does not appear to be a risk factor; however, personal history of NMSC is a risk factor with a relative risk of approximately 4.

Option D: A family history of melanoma in a second-degree relative is not a risk factor for melanoma, but melanoma in first-degree relatives at least doubles the risk of developing melanoma in a person.

24
Q

Ben, 10 years old, is brought by his mother to your GP clinic for advice on melanoma screening. Which one of the following puts Ben at the highest risk of developing melanoma?

A. Family history of melanoma in first-degree relatives.
B. Red hair.
C. Freckles.
D. Multiple sunburns.
E. Outdoor activities.

A

A. Family history of melanoma in first-degree relatives.

The following table classifies risk factors for developing melanoma in descending order. [SEE PAGE 207. Can only insert one photo.]

The following table classifies risk factors for melanoma as high, moderate, and lower than average.

Of the given options, a family history of melanoma carries the most significant risk (RR of at least 2), followed by red hair/freckling and then multiple sunburns, and prolonged sun exposure caused by outdoor activity.

25
Q

The following photograph shows skin lesions on the hand of a 73-year-old woman, who resides in a nursing home. The lesions are severely itchy and she keeps scratching them, especially in the night. The itch prevents her from a good night sleep. On examination, she looks disheveled and poorly cared for. Which one of the following would be the most appropriate treatment for her?

A. Permethrin 5% cream.
B. Itraconazole cream.
C. Topical corticosteroids.
D. Gamma benzene hexachloride.
E. Topical erythromycin.

A

A. Permethrin 5% cream.

The itching, the appearance of the lesions and living in a nursing home all favor of scabies as the most likely diagnosis. First-line medication for treatment of scabies is permethrin 5% cream. If treatment with permethrin fails or the patient is sensitive to this drug, benzyl benzoate can be used as second-line therapy. Benzyl benzoate is more irritating to skin and the treatment is likely to fail due to inadequate adherence.

Option B: Itraconazole cream is used for treatment of fungal skin infections. It has no role in treatment of scabies.

Option C: Topical corticosteroids of moderate potency may be used in conjunction with permethrin or benzyl benzoate, with no effect on the scabies.

Option D: Gamma benzene hexachloride (Lindane®) was formerly used for treatment of scabies. It is no longer approved for this purpose in Australia, as well as many other parts of the world.

Option E: If a bacterial infection superimposes scabies, the antibiotics of choice would be topical mupirocin or oral cephalexin in more severe cases. Erythromycin is not routinely used for.

26
Q

A 50-year-old woman presents to your clinic with multiple lumps. She has had these lumps for 20 years. The only complaint, apart from her concerns about the look of the lumps, is that they become painful at times. On examination, about 30 mobile subcutaneous lumps feel soft, rubbery, and lobulated when palpated. They are distributed bilaterally in her upper and lower extremities, chest, and back. She mentions that her mother has the same problem. She is otherwise healthy. Which one of the following is the most likely diagnosis?

A. Adipose dolorosa.
B. Multiple desmoid tumors.
C. Multiple desmoid cysts.
D. Multiple symmetrical subcutaneous lipomas.
E. Neurofibromatosis type I (Von Recklinghausen disease of nerve).

A

D. Multiple symmetrical subcutaneous lipomas.

The clinical findings such as mobility, subcutaneous location, and the consistency of the lumps make the multiple symmetrical lipomas the most likely diagnosis. Being lobulated is another important clue, as is the positive family history.

Lipomas are benign in many other sites. They are quite common and can be found in subcutaneous tissue. Although the subcutaneous fat layer is the most common site for lipomas to arise, they may be subfascial, intramuscular, or be found in many other sites. There is usually a genetic predisposition.

Option A: Adipose Dolorosa (Dercum disease) is a syndrome characterized by diffusely painful subcutaneous fat deposition without focal discrete lumps. This syndrome is most common in middle-aged women. The painful fat deposits are mostly confined to and seen in the abdomen and thighs.

Option B: Desmoid tumors arise from the deeper layers of the abdominal wall.

Option C: Epidermoid cysts contain keratinized material and are confined to the dermis. They are invariably attached to the overlying skin and are not mobile. They have a central punctum and may become infected and form abscesses. Like lipomas, epidermoid cysts do not progress to malignant lesions.

Option E: Neurofibromatosis type I, also called Von Recklinghausen disease, is a disease of nerves presenting with café-au-lait spots and pedunculated and sessile skin lesions (molluscum fibrosom). Given the history and characteristics of the lesions, neurofibromatosis is unlikely to be the diagnosis.

27
Q

The lesion shown in the following photograph is found on the back of a 50-year-old man. It has been present for the past 8 years and has grown slowly. Which one of the following is the most appropriate next step in management?

A. Review in 12 months.
B. Resection.
C. Excisional biopsy with 2mm margins.
D. Radiotherapy.
E. Topical imiquimod.

A

C. Excisional biopsy with 2mm margins.

A changing mole, especially this one with color variegation evident in the photograph, should be considered melanoma until proven otherwise. Even when a clinical diagnosis of melanoma is made with certainty, an excisional biopsy should be performed. This confirms the diagnosis and allows further planning of definitive treatment in terms of the width and orientation of excision margins, and whether or not a sentinel lymph node (SLN) biopsy is performed.

Immediate wide excision with margins based on a clinical estimate of tumor thickness may result in inadequate or excessive tumor clearance. It may also compromise subsequent management by making it impossible to perform accurate lymphatic mapping to identify draining lymph node fields and SLNs within those fields.

The Clinical Practice Guidelines for the Management of Cutananeous Melanoma in Australia and New Zeland, endorsed by the Australian National Health and Medical Research Council (NHMRC), recommend excision biopsy with 2 mm margins whenever possible.

Partial biopsies such as punch biopsies, incisional biopsies and shave biopsies are frequently unsatisfactory and may result in misdiagnosis due to unrepresentative sampling. However, an incision, punch or shave biopsy from the most suspicious area of a large pigmented lesion may be appropriate when complete excision is difficult.

28
Q

The lesion shown in the accompanying photograph has been present on the finger of a 56-year-old woman for the past 8 months. It has slowly enlarged since then and is causing mild discomfort. Which one of the following is the most likely diagnosis?

A. Chronic paronychia.
B. Heberden’s node of osteoarthritis.
C. Pyogenic granuloma.
D. Mucous (synovial) cyst of the finger.
E. Rheumatoid nodule.

A

D. Mucous (synovial) cyst of the finger.

The shiny nodule shown in the photograph which appears to be fluid-filled is suggestive of the digital synovial cyst (mucous cyst of the finger ).

Mucous (synovial) cysts of the finger are subcutaneous cystic lesions found on the dorsal aspect of the distal phalanx, distal to the distal interphalangeal joint, and overlying the germinal nail bed. They may cause distortion of the nail growth. They are located in the midline or laterally. They grow slowly and may undergo spontaneous resolution. Recurrence is likely if they are inadequately excised.

Option A: Chronic paronychia is nail fold inflammation usually caused by staphylococcus aureus or fungi. The characteristic feature is a painful swelling and erythema of the whole nail fold (crescent-shaped).

Option B: Heberden’s node of osteoarthritis is a bony lesion over the distal interphalangeal joints.

Option C: Pyogenic granuloma is a fleshy red vascular structure usually followed by minor trauma to the fingers. It is painless.

Option E: Rheumatoid nodules are periarticular nodules seen in patients with rheumatoid arthritis.

29
Q

Which one of the following statements is correct about lipomas?

A. Lipoma is a premalignant lesion.
B. Lipomas are usually lobulated.
C. Lipomas cannot be tethered to the skin.
D. Lipomas often occur in the scrotum.
E. Lipomas are invariably subcutaneous.

A

B. Lipomas are usually lobulated.

Lipomas are common benign tumors of mature fat cells that can be seen in any site of the body containing fatty tissue. They are not premalignant (option A).

Although they frequently occur in the subcutaneous fat tissue, it is not uncommon to see lipomas in the deeper layers such as beneath the fascia or within muscles. Therefore, they are not invariably subcutaneous (option E) deeper layers as well.

As there is very little fat in the scrotum, lipomas almost never occur there (option D), but fat deposition within the spermatic cord may be seen. Unlike lipomas, epidermoid cysts of the scrotum are common.

Lipomas are usually freely mobile and not attached to the overlying skin; however, in areas such as the back of the neck or the trunk, where the skin has less mobility, they could be less mobile or even immobile. This fact makes (option C) incorrect.

A lipoma’s most characteristic physical feature is the lobulated contour, which can reliably differentiate it from an epidermoid cyst and its smooth contour.

30
Q

On a health check, a 75-year-old man is found to have a lesion on his left cheek. He mentions that he has had it for three years. The lesion is illustrated in the accompanying photograph. Which one of the following is the most likely diagnosis?

A. Benign nevus.
B. Seborrheic keratosis.
C. Bowen’s disease.
D. Lentigo maligna melanoma.
E. Pigmented basal cell carcinoma.

A

D. Lentigo maligna melanoma.

The irregular borders and different color shades of the lesion are suggestive of melanoma. Since the lesion has been stable for 3 years, lentigo maligna melanoma (Hutchinson melanotic freckles) is the most likely diagnosis.

Lentigo maligna melanoma is classically a pigmented macular skin lesion found on the sun-exposed facial skin of the elderly. The color varies from black to light tan and is often variegated.

Option A: Benign nevus tends to have smoother borders and more even coloration.

Option B: Seborrheic keratoses are well-defined lesions with surfaces ranging from velvety to verrucous. The color can vary from black to light tan. Color variegation is not usually a feature.

Option C: Bowen’s disease (squamous cell carcinoma in situ) is a scaly plaque most commonly in sun-exposed areas. The plaque of Bowen’s disease can be mistaken for psoriasis or dermatitis.

Option E: Basal cell carcinoma has different characteristic features such as being pearly, raised, and associated with telangiectasis in nodular BCC, pearly thready borders in superficial BCC, and scar-like appearance in morphoeic BCC, none of which are present here.

31
Q

A 45-year-old woman presents with a mole on the lower eyelid of her left eye as shown in the following photograph. Which of the following would be the most appropriate management?

A. Excision of the lesion under local anesthesia.
B. Review in one month.
C. Reassure that the lesion is benign.
D. Refer the patient to a plastic surgeon.
E. Apply topical imiquimod.

A

D. Refer the patient to a plastic surgeon.

The photograph shows an irregular pigmented lesion on the lower eyelid of the left eye. The appearance of the lesion is suggestive of melanoma and complete excision and histopathological studies are indicated. Due to the anatomical location of the lesion, excision should be performed by a plastic surgeon to avoid tissue destruction or loss of function in such an anatomically delicate area.

When melanoma is suspected, excisional biopsy should be done as soon as possible; therefore ‘review in one month’ (option B) and reassurance (option C) are incorrect answers. Imiquimod (option E) is not an appropriate management for melanoma.

32
Q

An 8-year-old boy is brought to your practice by his mother, who is concerned about a patch of hair loss on his head. According to the mother, the hairfall started 2 weeks ago and left the area bald. On examination, there is a 2x3cm patch of hair loss on the scalp. There is no scarring, inflammation, or flaking of the area. The region is completely bald with no hair. Which one of the following is the most likely diagnosis?

A. Psoriasis.
B. Tinea capitis.
C. Alopecia areata.
D. Trichotillomania.
E. Discoid lupus erythematosus.

A

C. Alopecia areata.

Sudden-onset patchy hair loss, no hair growth, and a normal-appearing scalp favor alopecia areata as the most likely diagnosis.

Alopecia areata is a term used for a hair disorder characterized by one or more discrete areas of hair loss. Although hair loss can occur anywhere in the body, hair loss of the scalp, eyebrow, or beard often brings the patient to medical attention.

The physiopathology of alopecia areata is an autoimmune inflammation of anagen hair bulbs leading to the cessation of hair growth, but not the destruction of the hair follicle. The etiology of this autoimmune reaction is unknown. Probably, a genetic factor is involved, as 20% of the patients have a positive family history of alopecia areata.

The most common presentation of alopecia areata is the appearance of one or many round-to-oval completely hairless patches. The presence of exclamation point hairs (i.e. hairs tapered near the proximal end) is pathognomonic but is not always found. No epidermal changes are associated with hair loss, and the scalp remains intact. Additionally, hair loss in other hair-bearing areas supports the diagnosis. A positive ‘pull test’ in the periphery of the hair loss patch indicates that the disease is active and more hair loss can be expected.

Nail involvement is seen in up to 50% of patients and is associated with poor prognosis. Nail pitting is the most common form of nail involvement but other abnormalities have been reported as well. Fingernails (rather than toenails) are predominantly affected.

The course of the disease is extremely variable. The patch(es) may resolve spontaneously, remain unchanged, or expand and coalesce with other patches to result in alopecia totalis (entire scalp) or alopecia universalis (entire body).

With a single patch of hair loss, there is a 33% chance of hair re-growth in 6 months which will increase to 50% in one year.

Poor prognostic factors for hair re-growth include the following:
* Younger age of onset (< 10 years of age)
* Alopecia totalis
* Peripheral rather than central hair loss
* Associated nail pitting
* Coincidental atopy
* Organ-specific autoimmune disease
* Persisted hair loss for many years

The chance of relapse is very high (86% in one study), although the time of relapse cannot be predicted.

Option A: Psoriasis is characterized by erythematous scaly plaques, usually with a shiny silver appearance. In this patient, the scalp is intact; therefore, psoriasis is not likely.

Option B: Tinea capitis is associated with flaking and hairs of different lengths and different stages of growth, often broken and damaged.

Option D: Trichotillomania is compulsive pulling out of the hair. Although the scalp appears normal, hairs of different lengths are observed.

Option E: Discoid lupus erythematosus would have inflammation of the scalp of the affected area and probably scarring.

33
Q

A 45-year-old woman presents to your clinic concerned about a solitary patch of hair loss sized 5x6 cm. On examination, the scalp of the affected area is completely hairless, but normal-appearing with no scarring, inflammation, or flaking. Which one of the following is the next best step in management?

A. Hydrocortisone 1% cream.
B. Intradermal injection of triamcinolone.
C. Scraping of the scalp and microscopy after preparation with KOH.
D. Reassure her that the condition is self-limiting and she will not go bald.
E. Clotrimazole cream.

A

B. Intradermal injection of triamcinolone.

Unaffected scalp skin and complete hair loss are suggestive of alopecia areata. Alopecia areata is used for a hair disorder characterized by one or more discrete areas of hair loss. Although hair loss can occur anywhere in the body, hair loss of the scalp, eyebrow, or beard often brings the patient to medical attention.

The physiopathology of alopecia areata is an autoimmune inflammation of anagen hair bulbs, leading to the cessation of hair growth, but not the destruction of the hair follicle. The etiology of this autoimmune reaction is unknown. Probably, a genetic factor is involved because 20% of patients have a positive family history of alopecia areata.

The course of the disease is highly variable. Hair growth may occur and patches resolve spontaneously; however, patches may remain unchanged, or expand and coalesce with other patches to result in alopecia totalis (entire scalp) or alopecia universalis (entire body).

With a single patch of hair loss, there is a 33% chance of hair re-growth in 6 months which will increase to 50% in 12 months.

Alopecia areata is managed as follows:
Intralesional or topical potent corticosteroids are the initial treatment for most patients with patchy alopecia areata.

Intralesional corticosteroid injection has a high probability of success. It is appropriate for eyebrows and small areas on the scalp, but not for wide or multiple scalp or eyelash involvement. Injection of corticosteroids for alopecia of the eyelashes can cause cataracts, glaucoma, and cutaneous atrophy. The two commonly used preparations for intradermal injections are:

  • Triamcinolone acetonide 10mg/ml
  • Betamethasone (acetate + sodium phosphate) 5.7mg/ml

The injections should be 6 weeks apart.

While potent topical corticosteroids are frequently used to treat alopecia areata, evidence for their effectiveness is limited. Topical corticosteroids are usually reserved as first-line therapy for children and adults who cannot tolerate intralesional injections.

The choices of topical therapy are outlined in the following table: —

For those with extensive lesions (>50% of scalp hair loss) or recurrences, topical immunotherapy is probably the most effective treatment and should be considered first-line. A potent contact allergen is applied to the scalp weekly to precipitate an allergic contact dermatitis. The consequent mild inflammatory reaction is associated with hair regrowth through an unknown mechanism.

For children younger than 10 years, minoxidil solution and/or topical corticosteroids are considered first.

Dithranol cream can be used for those chronic stable areas of hair loss that may have previously failed to respond to corticosteroid therapy.

Option A: Hydrocortisone 1% is a mildly potent topical corticosteroid. Mildly potent corticosteroids are not effective in the treatment of alopecia areata.

Option C: Scraping of the scalp and preparation with KOH for microscopy would be the correct answer if tinea capitis were suspected. Tinea capitis would have flaking and hairs of different lengths as other presenting symptoms in addition to patches of hair loss.

Option D: While in a good percentage of affected individuals alopecia areata can persist or even progress to alopecia totalis or universalis, reassurance is not an appropriate option.

Option E: Clotrimazole cream is used for fungal skin infections. It is not effective against alopecia areata. Even if tinea was the case, this option was incorrect because tinea of hair should be treated with oral antifungal agents.

34
Q

A 17-year-old boy presents to your practice with lesion shown in the accompanying photograph. The lesions have failed to improve with conventional therapy. Which one of the following would be the most appropriate management?

A. Doxycycline.
B. Hydrocortisone 1% cream.
C. Isotretinoin.
D. Benzoyl peroxide.
E. Occlusive makeup.

A

C. Isotretinoin.

The photograph shows severe cystic acne and scarring. Systemic isotretinoin (Raccutane®) is the treatment of last resort for severe cystic acne or acne with scarring if conventional therapy fails. In this patient with no response to conventional therapy, systemic isotretinoin is the most appropriate management option.

For mild papulopustular acne, the use of either a topical retinoid or benzoyl peroxide is recommended. If the treatment fails, topical clindamycin or erythromycin is added. Salicylic acid, 3% to 5% in 70% ethanol, can be used for mild truncal acne.

For moderate papulopustular acne +/- trunk involvement +/- nodules, not only the strength of topical retinoids or benzoyl peroxide should be increased, but oral rather than topical antibiotics should be used. Oral doxycycline is the first-line option. Tetracycline or minocycline can be used alternatively as second- and third-line options, respectively. Erythromycin (250-500mg, twice daily) is the choice if tetracyclines are not tolerated or contraindicated (e.g. in pregnancy).

For moderate to severe acne +/- nodules +/- cysts, topical adapalene or tretinoin plus antibiotics is used. If there is no improvement by 6 weeks or insufficient response by 12 weeks, the antibiotic is changed or the dose is increased. If such measures fail, referral for systemic treatment with isotretinoin is the most appropriate management.

35
Q

Which one of the following parts of the nail is more commonly affected by tinea unguium?

A. Nail fold.
B. Nail bed.
C. Nail plate.
D. Cuticle.
E. Entire nail including nail bed.

Anterior view of the nail unit.
A

C. Nail plate.

Tinea unguium is the infection of the fingernails or toenails caused by dermatophytes. Generally, fungal infection of the nails is termed onychomycosis (plural: onychomycoses). Onychomycoses which are specifically caused by dermatophytes are referred to as tinea unguium. There are three distinctive presentations:

  • Distal subungual
    This presentation is the most common form of fungal nail infection. The most common culprit to this presentation is Trichophyton rubrum. It begins with a whitish, yellowish or brownish discoloration of the distal corner of the nail, which gradually spreads to the entire width of the nail plate and then slowly extends toward the cuticle.
  • Proximal subungual
    The most common cause is Trichophyton rubrum. It presents with whitish, yellowish, and brownish discoloration of the nail, starting from around the cuticle and spreading distally.
  • White superficial
    The most common cause is Trichophyton mentagrophytes. It starts with a dull white spot on the surface of the nail plate. The whole nail plate may eventually be involved. The white areas are soft and can be scraped gently to obtain a sample.

In all three types, the nail plate is the often affected part.

Lateral view of the nail unit.
36
Q

A 15-year-old boy presents to your practice with a lesion shown in the following photograph. The lesion is itchy. Which one of the following is the most likely diagnosis?

A. Psoriasis.
B. Tinea of the face.
C. Seborrheic dermatitis.
D. Allergic reaction.
E. Tinea versicolor.

A

B. Tinea of the face.

The photograph shows a circular erythematous scaly lesion with a central clearing. This picture along with the itchiness as the main symptom is highly suggestive of tinea infection of the face (Tinea faciei) as the diagnosis.

Tinea is caused by dermatophytes - fungi that affect any part of the skin. The typical rash of tinea has the following characteristic features:
* Annular or arcuate (arc-shaped)
* Scaly and itchy
* Definite edges and central clearing as it expands

Option A: Psoriasis is a systemic disease. Although the rashes of psoriasis may resemble tinea at some stages, they tend to appear bilaterally and in a symmetrical fashion. In addition, silvery scaling is often present; however, scaling may be absent in some forms of psoriatic lesions. Central clearing is not a feature of psoriatic rash.

Option C: Dermatitis may present similar to tinea infection, but central clearing makes tinea a more likely diagnosis.

Option D: Allergic drug reactions can cause various clinical pictures such as urticaria, different types of rash, and dermatoses. The photograph however illustrates a typical case of tinea.

Option E: Tinea versicolor is a common skin fungal infection caused by Malassezia furfur. It presents with a well-demarcated macular rash that is hyperpigmented or hypopigmented and slightly itchy. It is more commonly seen in the upper trunk, however, it may involve the whole trunk, neck, and even upper limbs. Tinea versicolor lesions do not have central clearing.

37
Q

An 8-year-old boy is brought to your practice with a complaint of itchy lesions shown in the following. He has been recently prescribed cotrimoxazole. Which on of the following could be the most likely diagnosis?

A. Tinea infection.
B. Psoriasis.
C. Contact dermatitis.
D. Allergic drug reaction.
E. Tinea versicolor.

A

A. Tinea infection.

The photograph shows multiple round lesions of different sizes over the face, chest, abdomen, and arm. The lesions have prominent circular edges with centers being clear. With itchiness in the history, dermatophytic infection (tinea) is the most likely diagnosis.

Tinea is caused by dermatophytes fungi that affect any part of the skin. The typical rash of tinea has the following characteristic features:
* Annular or arcuate (arc-shaped)
* Scaly and itchy
* Definite edges and central clearing as it expands

Option B: Psoriasis is characterized by erythematous patches distributed symmetrically. A unique feature of psoriasis is the silvery scaling of the patches absent here. Moreover, psoriatic lesions do not have central clearing.

Option C: Contact dermatitis can cause an itchy scaly erythematous rash without central clearing.

Option D: Allergic reactions to the medication (e.g. cotrimoxazole) can cause various clinical pictures, including an itchy erythematous rash but without central clearing. Wheals (hives) have clear centers, but the lesion’s appearance differs from those shown in the photograph. There is usually no scaling.

Option E: Tinea versicolor presents with hypo/hyperpigmented macules of different sizes with slight scaling and itching. There is no central clearing of the lesions.

38
Q

A 23-year-old woman presents to your GP clinic with a rash on her back. The rash appeared two months ago as a 3x4 cm oval patch on her upper back that later on was followed by an eruption of smaller spots with occasional itching. The spots are paler than the surrounding skin. She has tried tanning oils and salons but the rash persists. On examination, there are multiple 3- to 5-mm macules on her back distributed in a ‘Christmas tree pattern. Which one of the following is the most likely diagnosis?

A. Lichen simplex chronicus.
B. Pityriasis rosea.
C. Allergic contact dermatitis.
D. Atopic dermatitis.
E. Tinea corporis.

Christmas Tree Pattern (from explanation)
A

B. Pityriasis rosea.

The clinical picture and the course of symptoms are highly suggestive of pityriasis rosea (PR) (fine pink scale) is a common skin disorder seen in otherwise healthy people, especially in children and young adults. The cause is thought to be a viral infection (possibly herpes virus 6 or 7).

The eruption begins with a ‘herald patch’ that may be mistaken for tinea. This rash is then followed in approximately 2 weeks by the development of multiple, scaly, salmon-colored macules, each about 1 to 2 cm in size and oval in shape. The eruption is symmetric and most commonly involves the chest, back, abdomen, and adjoining areas of the neck and limbs. Lesions are not usually seen on the face, hands, and feet. The lesions are arranged along the skin creases to form an appearance similar to a Christmas tree. The rash may be itchy.

Pruritus is commonly evident in 25-75% of patients and is usually of mild-to-moderate severity. Secondary eczematous changes can occur if pruritus is severe. A small number (approximately 5%) of patients with PR have mild prodromal symptoms (e.g. malaise, fatigue, headache, nausea, anorexia, chills, fever, and arthralgia) that precede the appearance of the herald patch. Lymphadenopathy may occur before the onset of the rash.

As a rule, PR only needs symptomatic management. Menthol is often effective for mild itching. Moderately potent corticosteroids are used for more severe cases of itching. Exposure to sunlight may be effective for the alleviation of itching as well as hastening the resolution of the rash.

The condition resolves spontaneously in 6-8 weeks in most cases, but some lesions may persist for as long as 3-4 months. With the resolution of the eruption, post-inflammatory pigment change can be observed. Both hypopigmentation and hyperpigmentation can follow the rash. Dark-skinned people may experience hyperpigmentation that takes a few months to resolve.

PR-like eruptions are differential diagnoses. This condition is associated with many drugs such as acetylsalicylic acid, barbiturates, bismuth, captopril, clonidine, gold, imatinib, isotretinoin, ketotifen, levamisole, and metronidazole, as well as vaccines such as BCG, HPV, and diphtheria.

39
Q

Jane, 21 years old, presents with a painless lesion on the distal interphalangeal joint of her right middle finger. The lesion has been present for the past 2 months and increased in size. The lesion is shown in the following photograph. On examination, she is otherwise healthy. Which one of the following is the most likely diagnosis?

A. Abscess.
B. Pyoderma gangrenosum.
C. Pyogenic granuloma.
D. Basal cell carcinoma.
E. Epidermoid cyst.

A

C. Pyogenic granuloma.

The fleshy red lesion on the distal interphalangeal joint of this patient is consistent with the diagnosis of pyogenic granuloma (PG). PG is a common benign acquired vascular neoplasm of the skin and mucous membrane. The etiology is unknown.

Some think it might be due to minor trauma, especially for PGs of fingers. Pyogenic granuloma is a misnomer because the lesion is neither pyogenic nor granuloma.

It is an erythematosus (fleshy red) dome-shaped papule or nodule that bleeds easily and is prone to ulceration, erosion, and crusting. It is most often seen in adolescents and young adults. Up to 5% of pregnant women may experience it.

Although very rare, PGs have been reported in the gastrointestinal tract, the larynx, nasal mucosa, conjunctiva, and cornea.

Option A, D, E: Abscess, epidermoid cyst, and mucous cyst are covered with skin.

Option B:Pyoderma gangrenosum typically presents with a well-defined border, which is usually violet or blue.

40
Q

A 45-year-old woman presents to your GP clinic for review. A few months ago, she developed redness around her nose and cheeks that became worse after she drank alcohol. Recently, she was asked, by one of her colleagues at work, if she has alcohol problems because her appearance resembles those with excessive alcohol use. She drinks an average of 10 units of alcohol per week. Her facial appearance is shown in the accompanying photograph. Which one of the following is the most likely diagnosis?

A. Seborrheic dermatitis.
B. Erysipelas.
C. Acne rosacea.
D. Systemic lupus erythematous.
E. Alcohol-related skin disease.

A

C. Acne rosacea.

The photograph shows an erythematous butterfly rash. Additionally, the chin is also involved. Several papules and pustules are noted as well. The clinical picture presented in the photograph, along with the history, is suggestive of rosacea (also known as acne rosacea) as the most likely diagnosis.

Rosacea is a common chronic disorder that mainly involves the face. It tends to present in middle-aged people but may start at earlier ages. It is more common in cold climates.

The condition often begins as an exaggerated or prolonged flushing tendency, with erythema affecting the central face or the butterfly area in particular. Sometimes erythema can be seen in the chin and forehead. Initially, the erythema occurs intermittently but later becomes persistent. Sometimes, rosacea is associated with edema. Telangectiasis is often present. Sterile inflammatory papules, pustules, and nodules may be present mimicking acne. The distinguishing feature is the absence of comedones.

Patients often complain that their faces feel hot, burn, sting or itch. The patient often reports that their face is increasingly easily irritated by topical products.

The etiology of rosacea is unknown. Alcohol is not a cause but it can trigger the flushing and worsen the symptoms. Other triggering factors include:
* Hot or cold temperatures
* Wind
* Hot drinks
* Caffeine
* Exercise
* Spicy foods
* Emotions
* Topical products that irritate the skin and decrease the barrier
* Medications that cause flushing

In about 50% of the patients, blepharoconjunctivitis is observed. It presents with itching, burning, grittiness or foreign body sensation in the eye and erythema and swelling of the eyelid. More advanced cases can develop enlarged sebaceous glands and connective tissue changes resulting in a bulbous, rhinophymatous nose.

Seborrheic dermatitis, SLE, and erysipelas are among the differential diagnosis but not consistent with the history. None of these conditions have alcohol as a triggering factor.

Option A: Seborrhoeic dermatitis has scaling of the skin as a prominent feature.

Option B: Erysipelas is associated with the abrupt onset of erythematous butterfly rash almost always caused by streptococcus pyogenes. There is often fever and constitutional upset. It is not a chronic condition and is not triggered by alcohol use.

Option D: Although the rash resembles that of SLE, the lack of other signs and symptoms favoring SLE makes it least likely.

Option E: Alcohol-related skin disease is a general term, not a diagnosis. Rosacea worsens by alcohol but is not caused by it.

41
Q

A 17-year-old girl presents to your GP practice with redness and swelling of the sun-exposed areas of her face after she spent 60 minutes in the sun. She is on treatment for acne. On examination, erythema and edema of the sun-exposed areas of her face, neck and upper chest are noted. There are bullae all over the area. Which one of the following is the most likely diagnosis?

A. Doxycycline phototoxicity.
B. Allergic contact dermatitis.
C. Erythromycin phototoxicity.
D. Stevens–Johnson syndrome.
E. Benzoyl peroxide phototoxicity.

A

A. Doxycycline phototoxicity.

The scenario describes a typical case of photosensitivity associated with medication, namely drug-associated photoeruption.

There are two basic types of photoeruptions, which differ in clinical appearance and pathogenesis:

Phototoxic eruption - by far, phototoxic eruptions are the most common drug-induced photoeruptions. They are caused by absorption of UV light by the causative drug resulting in cell damage. UVA is the most common wavelength implicated; UVB and visible light can elicit reactions with some drugs. The eruption is typically an exaggerated sunburn, often with blisters. The following drugs are the most common causes of phototoxic eruptions:
* NSAIDs
* Quinolones
* Tetracyclines (e.g.doxycycline)
* Sulfonamides
* Phenothiazines

Photoallergic eruptions— photoallergy is a lymphocyte-mediated reaction caused by exposure to UVA. It is postulated that the absorbed radiation converts the drug into an immunologically active compound that is then presented to lymphocytes, causing a spongiotic dermatitis (eczema). The eruption is characterized by widespread eczema in photo-exposed areas: face, upper chest, and back of the hands.

Most photoallergic reactions are caused by topical agents including biocides added to soaps (halogenated phenolic compounds) and fragrances such as musk ambrette and 6-methyl coumarin. Systemic photoallergens such as the phenothiazines, chlorpromazine, sulfa products, and NSAIDs can produce photoallergic reactions; however, most of their photosensitivity reactions are phototoxic.

The characteristic skin lesion in this patient (edema, erythema and blistering) suggests a phototoxic eruption to one of the medications she is on for treatment of acne. Of the options, doxycycline (a tetracycline) is most likely to have caused this presentation.

Option B: Allergic contact dermatitis, as the name implies, require skin contact with an specific allergen to trigger a cutaneous immune response, most often characterized by eczema (spongiotic dermatitis). Moreover, contact dermatitis is not associated with UV exposure.

Option D: Stevens – Johnson syndrome, sometimes called erythema multiforme major, is a hypersensitivity response that can be triggered by the same drugs mentioned above, but with a different presentation. Furthermore, UV exposure does not play a role. Stevens – Johnson syndrome presents with epidermal and muscosal necrosis and sloughing of the skin and mucous membrane.

Option C and E: Benzoyl peroxide and erythromycin have not shown proven association with photoeruptions.

42
Q

A 2-year-old boy is brought to the emergency department, by his mother, after he developed a rash following a bee sting 20 minutes ago. The appearance of the rash is shown in the following photograph. On examination, there is no breathlessness, wheezing, abdominal pain, or hemodynamic instability. Which one of the following is the most appropriate management?

A. Intravenous epinephrine.
B. Intravenous diphenhydramine.
C. Intramuscular epinephrine.
D. Oral promethazine.
E. Oral corticosteroids.

A

D. Oral promethazine.

The picture shows generalized urticaria (hives) of the trunk. The rash also has involved the buttocks and arms but not the eyelids. In approach to patients with immediate hypersensitivity, the initial assessment should be focused on airway, breathing and circulation (ABC).

In the presence of any of the following situations, intramuscular epinephrine is always the most appropriate immediate management:
* Any acute onset of hypotension or bronchospasm or upper airway obstruction, even if typical skin features are not present.
* Any acute-onset illness with typical skin features (urticarial rash or erythema/flushing, and/or angioedema) PLUS Involvement of respiratory and/or cardiovascular and/or persistent severe gastrointestinal symptoms.

NOTE - An adrenaline autoinjector (EpiPen® or Anapen®) may be used instead of an adrenaline ampoule and syringe. For children 10–20 kg (aged 1–5 years) EpiPen Junior® or Anapen Junior® should be used. Intravenous boluses of epinephrine are not recommended without specialized training, as they may increase the risk of cardiac arrhythmias.

In the absence of the above mentioned presentations, management revolves around treatment of urticaria (and not anaphylaxis). Oral antihistamines (e.g. promethazine) are mainstay of therapy in most cases. Intravenous route is used when the urticaria is severe or eyelids are involved. If there is no response to antihistamines, oral corticosteroids are considered.

43
Q

A 65-year-old man presents with an exquisitely painful vesicular rash, which has been present for the past 7 days and is increasingly painful. The rash is shown in the following photograph. His past medical history is unremarkable. Physical examination is otherwise inconclusive. Which one of the following is the most appropriate immediate management?

A. Oral famciclovir.
B. Intravenous famciclovir.
C. Oral amitriptyline.
D. Intramuscular immunoglobulin.
E. Oral Phenergan® (promethazine).

A

C. Oral amitriptyline.

The appearance of the rash and the history suggest herpes zoster (shingles) infection as the most likely diagnosis.

Herpes zoster (shingles) is caused by the reactivation of the varicella-zoster virus (VZV). It usually occurs in adults but can be seen in children and in the first 2 years of life if there has been a history of maternal varicella.

Pain is a significant complaint in patients with shingles. Tricyclic antidepressants and the anticonvulsant gabapentin are the most effective medications for pain control in neuropathic pain associated with shingles.

Option A and B: Antiviral agents such as famciclovir, valacyclovir, or acyclovir should be used in any patient seen within 72 hours of the onset of vesicles, all patients with ophthalmic herpes zoster, and in immunocompromised patients. For pregnant women, only acyclovir (first line) or valaciclovir (second line) are used because the safety of famciclovir during pregnancy has yet to be established.

Option D: Varicella zoster immunoglobulin (VZIG) and vaccine are used for prophylaxis. Their use is not effective in the treatment or for pain control after the infection has established.

Option E: Promethazine is not effective for pain control in shingles.

44
Q

A 60-year-old man presents to your clinic with complaints of fever and a painful swollen right thigh. He has diabetes well controlled on metformin 500mg 8-hourly and hypertension for which he is taking valsartan 80mg, daily. He is on atorvastatin 20 mg daily for hypercholesterolemia as well. His recent medical history is remarkable for deep vein thrombosis (DVT) of his right leg, for which he is currently on warfarin. One week ago, he was admitted to the hospital after one episode of syncope and started on amiodarone after he was diagnosed with ventricular tachycardia (VT). Physical examination is remarkable for a temperature of 38.3°C and a warm swollen tender right thigh that is erythematous. The right thigh circumference is 3 cm greater than that of the left thigh. Which one of the following could be the most likely cause of this presentation?

A. Deep vein thrombosis(DVT).
B. Cellulitis.
C. Drug interaction.
D. Hematoma.
E. Rhabdomyolysis.

A

B. Cellulitis.

At first glance, there are a number of differential diagnoses for a swollen painful thigh including DVT, cellulitis, and hematoma. This patient is on warfarin for treatment of a previous DVT, which is a risk factor for another venous thromboembolic event (VTE) such as another DVT or pulmonary embolism (PE). However, it is less likely, yet not impossible, for this patient to develop another VTE while on treatment with warfarin. Therefore, DVT of the right leg (option A) is a weak possibility.

The interaction between warfarin and amiodarone is well-known. Amiodarone results in the decreased metabolization of warfarin through hepatic pathways and leads to increased bleeding tendency. Considering that the patient has been recently started on amiodarone while on warfarin, a hematoma (option D) can also be a possibility. However, the presence of fever, warmth, and erythema of the swelling makes this diagnosis less likely. Pressure from a hematoma on surrounding tissues such as skin or deeper layers (e.g. muscles can compromise circulation and results in complications such as necrosis or a superimposed infection that can justify, to some extent, the clinical picture; however, the expected sequence would be hematoma formation first and then the development of signs such as tenderness, warmth, and erythema later. It is also worth reminding the fact that hematomas present as bruises or bluish not red discoloration. Hematomas are not warm and are often non-tender. Swelling is a feature though.

Drug interaction (option C) between warfarin and amiodarone can result in increased bleeding tendency and the development of a hematoma, and interaction between atorvastatin and amiodarone can lead to an increased potential for the development of statin-related muscle problems. None of these events are associated with the given presentation and are very unlikely to be the cause of this clinical picture.

Rhabdomyolysis (option E) is a serious condition followed by muscle fibers breakdown and release of muscle cell contents such as myoglobin and potassium. Myoglobinuria results in the deposition of myoglobin in the kidneys and renal failure. Release of excess potassium from damaged muscle cells leads to hyperkalemia and serious complications such as cardiac arrhythmias. Statins such as atorvastatin and simvastatin in particular are metabolized by cytochrome P450-3A4 (CYP3A4), and amiodarone is a potent inhibitor of this cytochrome. Concomitant use of amiodarone and statins can result in increased activity of statins and rises the likelihood of statin-related adverse effects such as muscular pain, myopathy, and rarely rhabdomyolysis. However, in the event of statin-induced rhabdomyolysis, a systemic presentation is expected. Rhabdomyolysis never causes the focal signs mentioned in the scenario.

Of the given options, this clinical picture is most consistent with cellulitis of the left thigh as the most likely diagnosis. Cellulitis presents with systemic signs such as fever and focal findings of induration, warmth, redness, and tenderness of the affected area. Diabetes can be a risk factor.

45
Q

John, 60 years of age, presents to your office because he is concerned about a skin lesion on his forearm. The dermatoscopic view of the lesion is shown in the following photograph. Which one of the following could be the most likely diagnosis?

A. Actinic keratosis.
B. Bowen’s disease.
C. Malignant melanoma.
D. Dermatosis papulosa nigra.
E. Seborrheic keratosis.

A

E. Seborrheic keratosis.

The photograph shows an oval brown plaque on a sun-exposed area in an old man. There is also keratotic plugging in the surface of the lesion (yellowish-colored). These features are characteristic of seborrheic keratosis as the most likely diagnosis.

Seborrheic keratosis (SK) is the most common benign skin tumor in older adults with the incidence increasing with age. It is typically oval or round and brown lesions mostly in sun-exposed areas with adherent greasy scales. The color may range from black to tan. Usually, SK has a velvety verrucous (warty) surface and appears stuck on. However, some lesions are so verrucous that may resemble warts.

PLEASE LOOK AT PAGE 233-235 FOR PHOTOS

Option A:Actinic keratosis is characterized by flat small scaly lesions on the sun-exposed area of old adults, especially the back of the hands. Lesions can be multiple. Actinic keratosis is precancerous with the potential to become squamous cell carcinoma.

Option B: Bowen’s disease, similar to AK, is seen in the sun-damaged area of skin in older adults and is precancerous. It is flat and scaly such as AK but is larger in diameter and tends to be thicker with well-demarcated borders.

Option C: Malignant melanoma commonly presents with a rather flat lesion of various colors and irregular borders and shapes. Although nodular melanomas can present with a pigmented dome-shaped lesion, the commonality of SK, the presence of keratotic plugs, and the lack of color variety make malignant nodular melanoma a far less likely diagnosis compared to SK.

Option D: Dermatosis papulosa nigra is a variant of SK consisting of multiple brown-black dome-shaped, small papules found in the face of young to middle-aged people with colored skin, especially black. This single lesion on the forearm of an old man is SK not its variant dermatosis papulosa nigra.

NOTE - Stucco keratosis is another variant of SK that consists of large numbers of superficial grey-to-light brown flat keratotic lesions usually on the tops of the feet, ankle, and the back of the hands and forearms.

46
Q

The 7-year-old Evie is brought to your general practice by her mother who is concerned about a rash she has developed and is shown in the following photograph. The rash appeared almost two months ago on her abdomen and has spread to the extent evident in the photo. There is no complaint of pain or itching, but a few lesions have ulcerated (one shown in the photograph). Evie is otherwise healthy and has not had any serious illnesses so far. Her growth charts are all normal. Physical examination is otherwise normal. She goes to school and has two younger brothers and sisters at home. Which one of the following is the correct advice to give?

A. She should be excluded from school until the resolution of the rash.
B. She can go to school but should use separate towel and other personal items.
C. She should be given oral acyclovir.
D. Cryotherapy with liquid nitrogen is the treatment of choice for the rash.
E. She should use topical steroids.

A

B. She can go to school but should use seprate towel and other personal items.

The lesions shown in the photograph are pearly dome-shaped papules with central umbilication characteristic of molluscum contagiosum as the most likely diagnosis. Molluscum contagiosum is a common viral infection of childhood caused by molluscipoxvirus a member of the poxvirus family.

The infection presents with firm, smooth, spherical papules that are pearly white and have a central dimple (umbilicus). Most papules range from 1 to 3 mm; however, lesions of up to 1-2 cm have been reported mostly due to coalescing smaller lesions.

Seeking medical attention can be due to concerns about the rash itself, or more frequently the development of eczema in the surrounding skin. The latter can make a diagnostic challenge because eczema can obscure the primary lesions.

Lesions can develop anywhere, but flexures and areas of friction are more frequently involved. Lesions may also occur in the anogenital area and are not usually associated with sexual abuse of the child. Involvement of the eyelid margins may lead to chronic conjunctivitis.

Molluscum contagiosum has a benign course and nature. Most of the patients experience the spontaneous resolution of the lesions within three to six months, but on occasion, it may take up to three years.

In most cases, active treatment is not recommended because spontaneous resolution usually occurs upon activation of the immune response which may take from three months to three years. For individual lesions, removal by cryotherapy (option D) or curettage may be considered but is not generally recommended as a routine treatment for the lesions.

Children with molluscum contagiosum do not require isolation or exclusion from school however (option A), it is better to avoid sharing towels or bathing together as these may increase the risk of spread to their brothers and sisters or other children.

Option C: Active topical or oral antiviral therapy, such as acyclovir, is unnecessary and recommended.

Option E: Short-term use of a topical steroid for the treatment of associated eczema is recommended. There is no associated eczema in the photograph, so topical steroids are not indicated.

NOTE - Inflamed lesions do not require antibiotics either orally or topically.

47
Q

A 51-year-old man presents to the hospital with a swollen hand shown in the following photograph. He was gardening in his courtyard yesterday when his left index finger was injured. This morning, he noticed that his left hand is tender, swollen, and red. On examination, he has a blood pressure of 134/78 mmHg, a pulse of 88, a respiratory rate of 16 breaths/min, and a temperature of 38.1°C. Which one of the following is the most appropriate next step in management?

A. Ultrasound of the hand.
B. MRI of the hand.
C. Admission to the hospital and commencement of intravenous antibiotics.
D. CT scan of the hand.
E. Discharge him home on oral antibiotics and review in 24-48 hours.

A

E. Discharge him home on oral antibiotics and review in 24-48 hours.

The swollen, red, and tender hand following a recent injury makes cellulitis the most likely diagnosis. Cellulitis is a clinical diagnosis and no imaging studies such as ultrasound (option A), MRI (option B), or CT scan (option D) are required.

Streptococcus pyogenes (S.pyogenes) or other Streptococcus species such as group B,C, or G are the most common causes of nonpurulent, recurrent (e.g., associated with lymphedema), or spontaneous rapidly spreading cellulitis. Staphylococcus aureus is less common for cellulitis and is often associated with penetrating trauma, ulceration, or an abscess. Purulent cellulitis is typically caused by Staphylococcus aureus.

Antibiotics are the mainstay of treatment. Oral antibiotics are adequate for cellulitis not associated with systemic features and are usually appropriate for patients with a single systemic feature of infection (e.g., fever alone).

For adults with two or more of the following clinical features, admission to the hospital and commencement of intravenous antibiotics (option C) should be used initially:
* Temperature more than 38°C or less than 36°C
* Heart rate of more than 90 bpm
* Respiratory rate of more than 20 breaths/minute
* White cell count more than 12x109/L or less than 4x109/L, or more than 10% immature (band) forms

Other indications for initial intravenous antibiotics (even without having 2 or more of the above) are:
* Intolerance of oral therapy
* Immunocompromised patients
* Comorbidities that could be associated with the rapid progression of cellulitis such as diabetes mellitus
* Early follow-up is not possible

Patients who are discharged on oral antibiotics require early assessment (within 24-48 hours) for treatment response.

For this patient with a fever of 38.1°C as the only concerning factor besides the cellulitis, discharging home on oral antibiotics with an early review is the most appropriate option to consider at this stage. Intravenous antibiotics should be considered next if the response to oral therapy is unsatisfactory.

Other advice to give this patient is rest and elevation of the affected area.

48
Q

A 27-year-old man presents with extremely itchy rash on his buttocks and elbows as shown in the following photograph. He has long-standing history of celiac disease. He was also diagnosed with major depression seven months ago for which he is under treatment with Zoloft. Which one of the following could be the most likely diagnosis?

A. Linear IgA dermatosis.
B. Neurotic excoriation.
C. SSRI-induced dermatitis.
D. Scabies.
E. Dermatitis herpetiform.

A

E. Dermatitis herpetiform.

The photograph shows papules and vesicles of which some are eroded. Presence of such features, the clinical picture, and the presence of long-standing celiac disease in history make dermatitis herpetiform the most likely diagnosis. Dermatitis herpetiform is a rare cutaneous condition presenting with persistent immunobulous disease with a strong link to celiac disease.

The condition is called herpetiform because blisters/vesicles develop in clusters resembling herpes simplex.

Dermatitis herpetiformis (aka Duhring-Brocq disease) is a rare skin condition presenting with persistent immune-related bullous formation. The name herpetiformis is derived from the tendency for blisters to appear in clusters, resembling herpes simplex. However, dermatitis herpetiformis is not due to viral infection.

Dermatitis herpetiform and celiac disease are linked as both are due to intolerance to the gliadin fraction of gluten. Gluten triggers production of IgA and an autoimmune response that targets the intestine and skin. Over 90% of patients with dermatitis herpetiform have gluten-sensitive enteropathy and 15-25% of patients with celiac disease have concurrent dermatitis herpetiform. These patients tend to have a more severe intestinal pathology in comparison to those without dermatitis herpetiformis.

Dermatitis herpetiformis symmetrically affects scalp, shoulders, buttocks, elbows, and knees. Lesions come in form of extremely itchy papules and vesicles on normal-looking or erythematous skin. Lesions form in clusters (herpes simplex-like). Vesicles are often unroofed and flat due to scratching by the patient.

Other forms in which dermatitis herpetiform can present are digital petechia or flat red patches, or wheels imitating other skin conditions such as scabies (option D), dermatitis, or linear IgA dermatosis (option A).

Linear IgA dermatosis (LAD) is an autoimmune subepidermal vesiculobullous disease that may be idiopathic or drug-induced. The clinical presentation is heterogeneous and appears similar to other blistering diseases, such as bullous pemphigoid and dermatitis herpetiform. The list of factors implicated in development of LAD continues to grow; however, vancomycin is the most documented agent causing LAD thus far.

Dermatitis herpetiform can cause hypopigmentation or hyperpigmentation after resolution.

In neurotic excoriation (option B), skin abnormalities are caused by traumatizing effects of scratching on skin. In this case presence of vesicles make such diagnosis unlikely.

SSRIs can cause an increase serotonin concentration in skin which results in allergic reaction presenting with pruritus and other skin abnormalities including macular rash, xanthoma, purpura, etc. However, given the history of celiac disease and the presence of papules and vesicles on elbows and buttocks make celiac disease more likely than SSRI-induced dermatitis (option C).

49
Q

A 61-year-old woman presents to your practice complaining of a rash on her left shin. On examination, there is a well-demarcated 3x2cm erythematous lesion. It is neither painful, nor itchy. The lesion is shown in the accompanying photograph. Which one of the following is the most likely diagnosis?

A. Seborrheic keratosis.
B. Actinic keratosis.
C. Invasive squamous cell carcinoma.
D. Psoriasis.
E. Bowen’s disease.

A

E. Bowen’s disease.

The large raised scaly plaque shown in the picture has the characteristic features of Bowen’s disease.

Bowen’s disease is squamous cell carcinoma in situ of the skin. The disease is more commonly seen in those over 60 years of age, but it can be found at any age (rare before 30 years). Sun exposure is the most significant risk factor.

Bowen’s disease presents as a well-demarcated erythematous and scaling plaque that may be mistaken for psoriasis or dermatitis. It is usually asymptomatic, but ulceration is a possibility. In 10-20% of cases there are multiple lesions. Although Bowen’s disease can virtually occur anywhere in the body, the sun-exposed areas, particularly the lower legs are the most common site of presentation.

The diagnosis is confirmed and documented by punch or shave biopsy. All of the following options can be successfully used for treatment with about the same efficacy and a recurrence rate of 5-10%:
* Topical fluorouracil
* Imiquimod
* Cryotherapy
* Curettage
* Cautery
* Excision
* Photodynamic therapy
* Laser therapy
* Radiation therapy

Mohs’ surgery is used when the anatomic location of the lesion is of functional consideration (e.g. penis, fingers) or for cosmetic issues. Occasionally, observation may be considered if the lesion is progressing slowly and is located in areas with poor healing ability such as the anterior leg of an old person.

50
Q

Jesse, 57 years old, presents to your GP clinic with concerns about a spreading brown spot on his left cheek. Physical examination suggests melanoma. Which one of the following is the most appropriate step in management?

A. Excisional biopsy.
B. Punch biopsy.
C. Referral to a plastic surgeon.
D. Radiotherapy.
E. Chemotherapy.

A

C. Referral to a plastic surgeon.

Based on current guidelines, any suspicious lesion for melanoma should undergo an initial excisional biopsy (option A) for confirmation. Other types of biopsies such as punch biopsy (option B) are neither as accurate nor recommended.

However, lesions in sensitive anatomical areas such as the face and neck should be biopsied by a plastic surgeon; therefore, referral to a plastic surgeon is the best action to take at this stage.

The initial management of melanoma is excision. Other modalities such as chemotherapy (option E) or radiotherapy (option D) might be considered later for advanced or metastatic melanomas, but not at this stage.

51
Q

Charlie, 65 years old, is concerned about recent changes in a mole on his chest. He had the mole for a long time but recently it has grown bigger. The mole is shown in the following photograph. Which one of the following is the most likely diagnosis?

A. Squamous cell carcinoma.
B. Superficial spreading melanoma.
C. Benign nevus.
D. Basal cell carcinoma.
E. Age-related skin changes.

A

B. Superficial spreading melanoma.

The photograph shows a pigmented lesion with color variation with recent growth. Of the options, superficial spreading melanoma is the most likely diagnosis.

Superficially spreading melanoma, is the most common type of melanoma that presents with an irregularly-pigmented plaque or macule. The first sign of melanoma is usually a new or existing mole or spot which changes in color, shape, or size.

Squamous cell carcinomas (option A) are usually flat and have no pigmentations.

Basal cell carcinomas (BCCs) have different forms of which nodular BCC is the most common. This type presents with a pearly nodule with telangiectasis. BCC is not usually pigmented.

Benign nevi (option C) often are of uniform color as do age-related pigmented skin lesions (option E).

52
Q

John, 42 years old, presents to your GP clinic to discuss skin cancer prevention. His past medical history is significant for a cutaneous squamous cell carcinoma (SCC) excised which was successfully treated years ago. He has fair skin with a tendency to tan rather than burn. He denies a history of sunburns as a child. Which one of the following is the most significant risk factor for developing another SCC?

A. Tendency to tan rather than burn with sun exposure.
B. History of SCC.
C. His age.
D. Absence of sunburns in childhood.
E. Fair skin.

A

B. History of SCC.

SCC is the second most common skin cancer in Australia after basal cell carcinoma. Risk factors for SCC are:
* Unprotected exposure to ultraviolet (UV) radiation from the sun or tanning bed
* Age over 50
* Male gender
* Fair skin (Fitz Patrick I and II)
* Immunosuppression
* Personal history of skin cancers (non-melanoma or melanoma)
* History of human papillomavirus (HPV) infection
* Precancerous skin lesions such as Bowen’s disease and actinic keratosis (AK)
* Chronic infections and skin inflammation from burns, scars, and other conditions

Of these risk factors, the presence of another SCC in history has the most significance (high risk). Fair skin (option E) is another high-risk factor for SCC but not as significant as the history of another SCC. In the absence of past history of skin cancer, it would be the correct answer.

He is under 50 and his age (option C) is not a risk factor for now. He tends to tan rather than burn (option A) which is a protective factor. While childhood sunburns are a risk factor, their absence (option D) is not one.

53
Q

A 43-year-old man presents with a painful rash on his back as shown in the following photograph. The rash erupted one week ago and was itchy and slightly sore. The soreness built up in intensity and now he has severe sharp pain. Which one of the following is the most appropriate treatment option for him?

A. Gabapentin.
B. Opioids.
C. Amitriptyline.
D. Systemic steroids.
E. Famciclovir.

A

D. Systemic steroids.

The rash presents a typical case of shingles caused by varicella zoster virus (VZV) infection.

Varicella zoster virus (VZV) infection causes two distinct clinical syndromes. Primary infection, also known as varicella or chickenpox, is a common pediatric viral disease. After a chickenpox infection, VZV remains latent in the neurons of cranial nerves and dorsal root ganglia. Subsequent reactivation of latent VZV can result in a vesicular rash in a dermatomal distribution, known as herpes zoster (shingles). The lesions are painful, and the pain may last long even after the resolution of the rash.

Pain that persists for 3 months or more is defined as post-herpetic neuralgia (PHN). The pain associated with acute zoster infection and PHN can be burning, sharp, stabbing, and constant or intermittent. More than 90 percent of patients with PHN also have allodynia, defined as pain evoked by normally nonpainful stimuli such as light touch.

Of note, pain management for acute zoster infection is different. For pain associated with acute herpes zoster infection, simple analgesia or systemic steroids are used as first-line options. The tricyclic antidepressant (TCA) Amitriptyline or opioids (option B) may be considered as the second line.

On the other hand, pregabalin, gabapentin, TCAs, or lignocaine patches are preferred first-line options for PHN. Opioids and tramadol are used as second-line alternatives.

Antiviral agents such as famciclovir, valacyclovir, or acyclovir should be used in any patient seen within 72 hours of the onset of the rash, all patients with ophthalmic herpes zoster, and in immunocompromised patients. For pregnant women, only acyclovir (first line) or valaciclovir (second line) are used because the safety of famciclovir during pregnancy has yet to be established. Antivirals are often given for 7 days.

This patient has presented beyond the 72-hour window, and has no other indications for antiviral therapy, and giving him famciclovir (option E) is futile. For him, pain control is the most significant concern. Of the options, systemic steroids such as prednisolone (50mg daily for 7 days) are the most appropriate. Gabapentin (option A) or amitriptylin (option C) was the correct option if the case was PNH (pain persisting for 3 months or beyond.

54
Q

A 52-year-old man presents with intense pain in his chest wall for the past 4 months. The problem started with the eruption of a rash over the area. The rash resolved in 2 weeks, but the pain has persisted and increased in intensity. He describes the pain as constant, sharp, and unbearable, affecting his life. He has used Panadol and ibuprofen for pain control with minimal effect. He also has diabetes, hypertension, and stage III chronic kidney disease, and denies any pain or pins and needles elsewhere. Which of the following is the most appropriate option for him?

A. Gabapentin.
B. Tramadol.
C. Amitriptyline.
D. Systemicsteroids.
E. Pregabalin.

A

C. Amitriptyline.

The scenario represents post-herpetic neuralgia, defined as chronic pain associated with herpes zoster infection (shingles) caused by the varicella zoster virus.

Varicella zoster virus (VZV) infection causes two distinct clinical syndromes. Primary infection, also known as varicella or chickenpox, is a common pediatric viral disease. After a chickenpox infection, VZV remains latent in the neurons of cranial nerves and dorsal root ganglia. Subsequent reactivation of latent VZV can result in a vesicular rash in a dermatomal distribution, known as herpes zoster (shingles). The lesions are painful, and the pain may last long even after the resolution of the rash.

Pain that persists for 3 months or more is defined as post-herpetic neuralgia (PHN). The pain associated with acute zoster infection and PHN can be burning, sharp, stabbing, and constant or intermittent. More than 90 percent of patients with PHN also have allodynia, defined as pain evoked by normally nonpainful stimuli such as light touch.

Pain control in the acute phase of the infection is often achieved by simple analgesics and/or systemic steroids (e.g., prednisolone 50mg/d for 7 days). Opioids or amitriptyline can be used as second-line options if adequate pain control is not achieved.

For PHN, the anticonvulsants gabapentin or pregabalin, tricyclic antidepressants (TCAs), or lignocaine dermal patches are used as first-line options. Of the TCA, amitriptyline is the most effective and commonly prescribed one.

With pain lasting for 4 months, this patient has PHN. Gabapentin (option A) and pregabalin (option E) are eliminated by renal excretion and should be used with caution and meticulous dose adjustment in patients with renal insufficiency and avoided if better first-line options are applicable. For him, amitriptyline is the most appropriate option for pain control.
TCAs inhibit the reuptake of norepinephrine and serotonin in the central nervous system. They are thought to increase the inhibition of nociceptive signals from the periphery. The preferred tricyclic drug for PHN is amitriptyline, starting at 10 mg each night and titrating slowly as tolerated, with a maximum dose of 150 mg daily.

Amitriptyline is excreted via the liver and its metabolism and is the most appropriate option in this scenario. Its anticholinergic adverse effects such as sedation and dry mouth) may limit the patient’s compliance.

Opioids (option B) or tramadol can be used as second-line agents, in those with contraindications to the first-line options.

While steroids (option D) are used for treating acute herpes zoster pain where inflammation seems to have a role, they have no effect on PNH as it has different pathophysiology.